2014 dxdy logo

Научный форум dxdy

Математика, Физика, Computer Science, Machine Learning, LaTeX, Механика и Техника, Химия,
Биология и Медицина, Экономика и Финансовая Математика, Гуманитарные науки


Правила форума


В раздел Пургаторий будут перемещены спорные темы (преимущественно псевдонаучного характера), относительно которых администрация приняла решение о нецелесообразности продолжения дискуссии.
Причинами такого решения могут быть, в частности: безграмотность, бессодержательность или псевдонаучный характер темы, нарушение автором принципов ведения дискуссии, принятых на форуме.
Права на добавление сообщений имеют только Модераторы и Заслуженные участники форума.



Начать новую тему Ответить на тему На страницу Пред.  1, 2, 3, 4  След.
 
 Re: Простая задачка по квантовой механике
Сообщение01.02.2017, 22:29 
Заслуженный участник
Аватара пользователя


30/01/06
72407
Ух как! Теперь у нас научные вопросы решаются криком? :-)

 Профиль  
                  
 
 Re: Простая задачка по квантовой механике
Сообщение01.02.2017, 23:32 
Заслуженный участник
Аватара пользователя


28/04/16
2388
Внутри ускорителя
Founder_Q в сообщении #1189151 писал(а):
Пришлось мне начать восстанавливать справедливость, хотя уже все это надоело, если честно..

Что-то Вы, ув. Founder_Q, как-то хреново справедливость восстанавливаете. :facepalm:

В случае, если кто-то в научном сообществе не согласен с результатами некоторой опубликованной статьи, то он просто пишет маленькую статью-комментарий на неё (называют подобные статьи как "комментарий на статью ... таких-то авторов, и выходные данные статьи"). В этой работе он тыкает указывает на ошибки и огрехи, сделанные в обсуждаемой работе, и (если может) приводит правильный результат. После, он посылает эту статью в тот же журнал, где она проходит стандартную процедуру peer review и в итоге публикуется в журнале. В особо запущенных случаях, можно добиться даже отзыва изначальной статьи...
Пример описанного выше: оригинальная статья и комментарий к ней. :lol:

Вы же вместо этого пишете полотна ( :? ) на форуме ( :facepalm: ), да такие, что их объём существенно превосходит размер необходимого типа статей... :roll: А сил, потраченных Вами на их написание, наверняка хватило бы на парочку таких комментариев в журнале. :lol1:


Founder_Q в сообщении #1189151 писал(а):
ЛЮБЫЕ ВОЛНОВЫЕ ФУНКЦИИ, КАК РЕШЕНИЯ РАЗЛИЧНЫХ ВАРИАНТОВ КВАНТОВОМЕХАНИЧЕСКОГО ДИФФЕРЕНЦИАЛЬНОГО УРАВНЕНИЯ (Шредингера) – ВСЕГДА ПРЕДСТАВЛЯЮТ СОБОЙ НЕПРЕРЫВНО-ДИФФЕРЕНЦИРУЕМЫЕ ФУНКЦИИ , (т.е. ВОЛНОВЫЕ ФУНКЦИИ – НЕ МОГУТ ИМЕТЬ НИ СКАЧКОВ , НИ ИЗЛОМОВ,

Вам бы тогда посмотреть на дифференцируемость волновых функций хотя бы в случае знаменитой "частицы в бесконечном ящике" (т.е. с потенциалом $V(x)=\begin{cases}
0,&\text{если $x \in (-a; a)$;}\\
\infty, & \text{если $x \in  (-\infty; -a] \cup [a, \infty) $.}\\
\end{cases}$ ) на границах ящика (т.е. при $x=\pm a$). Ну или возьмите произвольную задачу с кусочным потенциалом и посмотрите на то, как ведёт себя волновая функция в точках разрыва потенциала. :roll:

 Профиль  
                  
 
 Re: Простая задачка по квантовой механике
Сообщение02.02.2017, 00:55 
Заслуженный участник
Аватара пользователя


31/01/14
11053
Hogtown
madschumacher в сообщении #1189164 писал(а):
как-то хреново справедливость восстанавливаете

Какая справедливость, так и восстанавливает.

 Профиль  
                  
 
 Re: Простая задачка по квантовой механике
Сообщение02.02.2017, 01:59 


26/01/17

11
- Ну что тут можно еще сказать? - Писал я это все для аудитории, т.е. для Вас, ув. участники форума. Цель - просвещение аудитории и конструктивная критика/диалог. Судя по feedback на мои посты, ни того, ни другого, пока что, практически нет. Кроме того, хотелось прояснить для вас некоторые тонкие моменты, которые автор задачи, pvp - либо не понимает, либо сознательно "прячет под ковер". В итоге , pvp "продает" вам свои псевдонаучные измышления, содержащие просто неграмотные математические утверждения (на которые я также указал), вместо объективного анализа ситуации, который я тут провел.

Отвечая madschumacher: Никакую статью на эту тему я писать (пока) не планирую, слишком хлопотно, нудно и затратно по времени (напр. нужно будет нудно доказывать "нужность" такой публикации еще и для редакции, ведь такая статья - будет ударом и по их репутации). Кроме того, я считаю , что публиковать какую-либо статью-опровержение, не содержащую новых позитивных результатов - это несколько ущербная практика . Меня бы устроило, просто признание своих ошибок авторами этой статьи, т.к. все это на самом деле выросло из частной дискуссии с pvp по упомянутой статье, из-за его нежелания признавать очевидные вещи, и в т.ч. свои ошибки. Данный форум - просто еще одна публичная платформа для этой дискуссии.

Теперь, немного по сути некоторых последних реплик, по порядку:

g______d в сообщении #1189155 писал(а):

Я не понимаю, к чему вы стремитесь.


- Уже объяснил выше. К научной объективности . Даже на этом форуме.

g______d в сообщении #1189155 писал(а):
Свой уровень осведомлённости вы уже продемонстрировали (в сообщении, перенесённом в Пургаторий).


-Говоря об уровне осведомленности pvp, я не был голословен, мое утверждение было проиллюстрировано конкретными примерами (см.выше).

-Вам же, ув. g______d, даже если вы математик, (а если физик т о тем более), должно быть стыдно, что Вы не знаете того простого факта, что решения квантовомеханических задач, имеющие вероятностный смысл - должны представлять собой гладкие (или - что то же самое, непрерывно-дифференцируемые) функции . А вы писали мне в своем комментарии : ".. если потребовать непрерывность решения по времени, то единственным решением задачи будет указанное в первом посте.." - обращаю внимание всех участников: g______d здесь требует ТОЛЬКО непрерывность, а не непрерывную дифференцируемость решения и считает, что этого вполне достаточно для волновой функции! -Вы что же, g______d, где-нибудь видели волновые функции с "изломами" (т.е. НЕ непрерывно-дифференцирумые, со скачком первой производной)? - Покажите, если видели, и мы вместе посмеемся над этим. - И этот, с позволения сказать, "заслуженный комментатор" - еще посмел удалить мой пост, не понимая достаточно очевидных вещей из квантовой механики.

Далее, из той же оперы:

g______d в сообщении #1189155 писал(а):
УШ с разрывной правой частью в принципе не может иметь непрерывно дифференцируемого решения в классическом смысле (поскольку утверждение о том, что производная решения разрывна, является частью уравнения).


- Совершенно верно, я об этом и пишу здесь все время. Именно поэтому , а также потому , что в задаче имеется в виду временная динамика квантовой системы, которая всегда - непрерывно-дифференцируемая (т.е. плавная) - существование точного решения этой задачи привело бы к противоречию: если бы точное решение существовало, оно, математически, не должно было бы быть непрерывно-дифференцируемым (как Вы правильно пишете), а с квантово-механической точки зрения - наоборот, оно должно быть "плавным" (т.е. непрерывно-дифференцируемым, без разрыва своей временной производной). Это противоречие и означает, как раз, что точного решения эта квантово-механическая задача не имеет. У нее есть только приближенное, по теории возмущений, решение , которое я указывал ( и которое, именно в силу своей приближенности, может быть непрерывно-дифференцируемым, как того требует квантовая механика).
g______d в сообщении #1189155 писал(а):
С другой стороны, у этого уравнения есть каноническое и единственное решение в обобщенных функциях, которое и получается с помощью склеивания решений на двух интервалах с условием непрерывности. Это лучшее, что можно получить, решая это уравнение.

-Из написанного выше следует, что такое решение в обобщенных функциях - не имеет в данном случае никакого физического смысла. Кроме того, как быть с расходимостью неопределенности в разложении Дайсона (на которую я также указывал выше), вне теории возмущений? - Если Вы согласны с этой расходимостью, то Вы должны признать , что никакого (точного) "единственного решения в обобщенных функциях" для этой задачи не существует, а есть только бесконечный ряд Дайсона, который в этом случае (ступенчатой подынтегральной функции, с большой высотой скачка) -не сходится ни к какому конкретному пределу (т.е. - точное решение - не определено, как я и писал выше).

g______d в сообщении #1189155 писал(а):
Разговоры о физическом смысле в данном случае нужно было проводить до того, как уравнение было написано.


- Ну это вообще не ко мне, это, как раз, к pvp. :))

g______d в сообщении #1189155 писал(а):
Знают. Если бы не знали, было бы 2 уравнения, при $t<t_0$ и при $t>t_0$, и тогда была бы неопределённость в граничных условиях при $t=t_0$ справа и слева.


- ?? А что разве этой неопределенности "в граничных условиях справа и слева" здесь нет? - Если так, то каким образом связаны между собой пределы функции $B(t)$ , при стремлении $t$ к $t_0$ "слева" (это $1$) и "справа" (это произвольное число $A$) ? - Если же, g______d, Вы имеете в виду, граничные условия на решения двух получившихся уравнений Шредингера с различными постоянными гамильтонианами, при $t=t_0$, то кто Вам дал право "сшивать" 2 разных решения , отделенных друг от друга временным интервалом (скачка) $\delta t=(t_1-t_0)$, на котором у системы даже гамильтониан не определен? - Куда Вы дели эволюцию системы на интервале скачка, равном $\delta t=(t_1-t_0)$?
- Перечитайте еще раз те места в моих постах, где я пишу о невозможности построить точный оператор эволюции квантовой системы в случае когда в ее временной эволюции имеют место квантовые скачки или какие-либо проективные измерения ее наблюдаемых.

 Профиль  
                  
 
 Posted automatically
Сообщение02.02.2017, 02:06 
Заслуженный участник


09/05/12
25179
 i  Тема перемещена из форума «Помогите решить / разобраться (Ф)» в форум «Дискуссионные темы (Ф)»
Причина переноса: давайте-ка переедем сюда. Пока.

 Профиль  
                  
 
 Re: Простая задачка по квантовой механике
Сообщение02.02.2017, 02:16 
Заслуженный участник
Аватара пользователя


08/11/11
5940
Founder_Q в сообщении #1189182 писал(а):
обращаю внимание всех участников: g______d здесь требует ТОЛЬКО непрерывность, а не непрерывную дифференцируемость решения и считает, что этого вполне достаточно для волновой функции! -Вы что же, g______d, где-нибудь видели волновые функции с "изломами" (т.е. НЕ непрерывно-дифференцирумые, со скачком первой производной)? - Покажите, если видели, и мы вместе посмеемся над этим.


А вы не поленитесь, посчитайте частицу в ящике, как подсказывает madschumacher. Хоть с конечными стенками, хоть с бесконечными.

 Профиль  
                  
 
 Re: Простая задачка по квантовой механике
Сообщение02.02.2017, 02:26 
Аватара пользователя


11/06/12
10390
стихия.вздох.мюсли
Founder_Q в сообщении #1189182 писал(а):
Никакую статью на эту тему я писать (пока) не планирую, слишком хлопотно, нудно и затратно по времени
madschumacher в сообщении #1189164 писал(а):
Вы же вместо этого пишете полотна ( :? ) на форуме ( :facepalm: ), да такие, что их объём существенно превосходит размер необходимого типа статей

 Профиль  
                  
 
 Re: Простая задачка по квантовой механике
Сообщение02.02.2017, 02:33 


26/01/17

11
madschumacher в сообщении #1189164 писал(а):
Вам бы тогда посмотреть на дифференцируемость волновых функций хотя бы в случае знаменитой частицы в потенциальном ящике... Ну или возьмите произвольную задачу с кусочным потенциалом и посмотрите на то, как ведёт себя волновая функция в точках разрыва потенциала.


-Во-первых, во всех этих случаях волновая функция - все же непрерывно-дифференцируема во всей области, где она определена (т.е. она непрерывна по координате и имеет непрерывную первую производную по координате во всей этой области): см. например, в google что-нибудь вроде: "Одномерный потенциал с точкой разрыва. Непрерывность логарифмической производной волновой функции". Если же Вы имеете в виду очень специальный случай бесконечно-высоких стенок потенциальной ямы (который не реализуется здесь), т.е. иными словами, случай "жестких" граничных условий (когда и волновая функция и ее производная по координате на стенках такой ямы равны нулю ), то и в этом случае, очевидно , волновая функция , все же, является непрерывно-дифференцируемой функцией координаты во всей своей области определения (т.е. в самой потенциальной яме).

Ну и наконец, всегда есть разница между "граничными" и "начальными" условиями: все ,что Вы упомянули, это стационарные (не зависящие от времени) квантово-механические задачи с определенными (стационарными же, т.е. никак не зависящими от времени) граничными условиями, т.е. для них можно найти спектр постоянных собственных значений и т.п. В данном же случае речь идет о задаче существенно нестационарной, когда, в точке скачка по времени ,не определены не "граничные", а именно начальные условия (для второго этапа эволюции системы после скачка гамильтониана). В данном случае, в интервале скачка гамильтониана, у него вообще не существует никаких определенных собственных значений, т.к. и сам гамильтониан на интервале своего скачка - не определен.

 Профиль  
                  
 
 Re: Простая задачка по квантовой механике
Сообщение02.02.2017, 02:37 
Заслуженный участник
Аватара пользователя


04/09/14
5011
ФТИ им. Иоффе СПб
Уважаемый Founder_Q, давайте возьмем такую, вполне осмысленную, задачку (для простоты - одномерную). Летела себе частица. В момент времени $t_0$ нехороший человек возьми, да и стукни по ней молотком. Напишите пожалуйста классическое уравнение Ньютона для такой задачи (удар идеальный, т.е. время соударения - ноль). А если с этим справитесь, то тоже самое для квантовой частицы. Только слов не надо, пишите формулы.

 Профиль  
                  
 
 Re: Простая задачка по квантовой механике
Сообщение02.02.2017, 02:51 
Заслуженный участник
Аватара пользователя


31/01/14
11053
Hogtown
Многоуважаемый Founder_Q,
Пожалуйста, сообщите нам, в какой монографии или каком учебнике Вы вычитали, что волновая функция должна быть хотя бы непрерывной? Если мне не изменяет память, априори она принадлежит $L^2$ и не более того. Может быть это новое слово в науке?

 Профиль  
                  
 
 Re: Простая задачка по квантовой механике
Сообщение02.02.2017, 02:52 
Заслуженный участник
Аватара пользователя


08/11/11
5940
Founder_Q в сообщении #1189190 писал(а):
-Во-первых, во всех этих случаях волновая функция - все же непрерывно-дифференцируема во всей области, где она определена (т.е. она непрерывна по координате и имеет непрерывную первую производную по координате во всей этой области):


Второй производной уже нет, а уравнение по $x$ второго порядка, поэтому не хватает и приходится точно так же склеивать (или решать в обобщённых функциях).

Ну и вообще, есть модель Кронига-Пенни (расчёска $\delta$-функций).

 Профиль  
                  
 
 Re: Простая задачка по квантовой механике
Сообщение02.02.2017, 03:03 
Заслуженный участник
Аватара пользователя


28/04/16
2388
Внутри ускорителя
Founder_Q в сообщении #1189190 писал(а):
т.е. иными словами, случай "жестких" граничных условий (когда и волновая функция и ее производная по координате на стенках такой ямы равны нулю ),

А Вы не потрудитесь и посчитайте производную на стенке!

(спойлер)

у нас дифур 2-го порядка $\Rightarrow$ нужны 2 условия для задачи Коши. Мы берём в качестве их равенство в.ф. нулю на стенках (их 2), т.е. на производные мы не накладывали ограничения. Ну и когда считаем 1-ю производную, то она (внезапно!) не нулевая!!! :lol:

Founder_Q в сообщении #1189190 писал(а):
во всей своей области определения (т.е. в самой потенциальной яме)

Ну щас!! У в.ф. в этом случае область определения, насколько я помню, $\mathbb{R}$ :-) Просто за пределами ямы $\psi = 0$, поэтому и не интересно о ней говорить. :lol:

 Профиль  
                  
 
 Re: Простая задачка по квантовой механике
Сообщение02.02.2017, 03:07 
Аватара пользователя


11/06/12
10390
стихия.вздох.мюсли
madschumacher в сообщении #1189197 писал(а):
А Вы не потрудитесь и посчитайте производную на стенке!
madschumacher, за что вы его так. Пусть сначала посчитает производную за столом, сидя в удобном кресле. На стенке знаете, как неудобно писать. Тем более на шведской.

 Профиль  
                  
 
 Re: Простая задачка по квантовой механике
Сообщение02.02.2017, 03:27 


26/01/17

11
amon в сообщении #1189191 писал(а):
Летела себе частица. В момент времени $t_0$ нехороший человек возьми, да и стукни по ней молотком. Напишите пожалуйста классическое уравнение Ньютона для такой задачи (удар идеальный, т.е. время соударения - ноль)..


- Классика: Время соударения - ноль при конечном импульсе передачи (от молотка - частице) -означает бесконечную силу в правой части уравнения Ньютона - динамический метод здесь не подходит. Как решать: пишете два закона сохранения: энергии и импульса - зная начальные условия (импульс частицы и импульс молотка в начале всей истории) - получим то, что будет в конце :?) Детали динамики (т.е. самого процесса передачи импульса от молотка - частице) здесь не могут быть найдены, вследствие бесконечно малого времени соударения (да эти детали и не важны в данном случае)

amon в сообщении #1189191 писал(а):
.. то тоже самое для квантовой частицы...


?? В смысле: классическое уравнение Ньютона для квантовой частицы? Вы ничего не путаете, уважаемый? )))


- Квантовый аналог: Квантовым аналогом этой классической задачи являются всевозможные задачи рассеяния: пишется нестационарное уравнение Шредингера с интересующим исследователя потенциалом, на его решение (волновую функцию) накладываются граничные условия на бесконечности (обычно это плоские волны в асимптотике); дальше, по теории возмущений, (как и в рассматриваемом случае малого скачка потенциала) , рассматривая потенциал рассеяния как малое возмущение свободного гамильтониана, находятся эффективные коэффициенты отражения и прохождения падающих волн (собственных функций свободного гамильтониана), как функции их энергии , в различных приближениях по малому рассеивающему потенциалу.

Первый порядок такой теории возмущений - меняет фазу рассеяния - более-менее соответствует формулам pvp и называется в литературе 1-м борновским приближением, (можете свериться с учебниками), но будут также давать вклад и другие, высшие порядки ряда теории возмущений. Если возмущающий (рассеивающий) потенциал мал - все сходится, если не мал, как правило, все расходится. В самом общем случае произвольной величины рассеивающего потенциала отсуммировать весь ряд Дайсона (или, что то же самое , точно решить интегральное уравнение Липпмана-Швингера ) - как правило не удается. Поэтому в пределе очень большого рассеивающего потенциала - люди просто берут (если это не лишает смысла рассматриваемую ими задачу) уже совсем другое приближение (как правило, слабого туннелирования через "уже-не-рассеиватель", а "туннельный барьер"), но это уже совсем другая история..

 Профиль  
                  
 
 Re: Простая задачка по квантовой механике
Сообщение02.02.2017, 03:33 
Аватара пользователя


11/06/12
10390
стихия.вздох.мюсли
Founder_Q в сообщении #1189201 писал(а):
Вы ничего не путаете, уважаемый?
Очень характерный речевой признак, позволю себе заметить. Не «уважаемый имярек», а просто «уважаемый»; маскировка неуважения под уважение.

 Профиль  
                  
Показать сообщения за:  Поле сортировки  
Начать новую тему Ответить на тему  [ Сообщений: 51 ]  На страницу Пред.  1, 2, 3, 4  След.

Модераторы: photon, whiterussian, profrotter, Jnrty, Aer, Парджеттер, Eule_A, Супермодераторы



Кто сейчас на конференции

Сейчас этот форум просматривают: нет зарегистрированных пользователей


Вы не можете начинать темы
Вы не можете отвечать на сообщения
Вы не можете редактировать свои сообщения
Вы не можете удалять свои сообщения
Вы не можете добавлять вложения

Найти:
Powered by phpBB © 2000, 2002, 2005, 2007 phpBB Group